LSAT and Law School Admissions Forum

Get expert LSAT preparation and law school admissions advice from PowerScore Test Preparation.

User avatar
 Dave Killoran
PowerScore Staff
  • PowerScore Staff
  • Posts: 5852
  • Joined: Mar 25, 2011
|
#80599
Setup and Rule Diagram Explanation

This is a Mapping—Supplied Diagram game.

The game provides a diagram that should be used for your main setup. In examining the diagram, note that the small squares in the middle of cities 1-2-3-4 and cities 3-4-5-6 are there so that cities that are diagonal from each other, such as 1 and 4 or 4 and 5, do not share a common border. This fact plays an important role when considering the fifth rule.

The first rule is a cleanup rule, one that limits the number of solutions in the game.

The second rule establishes that the maximum number of jails in a city is one, and the maximum number of hospitals in a city is also one.

The third rule is a negative grouping rule, and it establishes that jails and universities do not appear in the same city:
PT9-Oct1993_LGE-G4-srd1.png
The fourth rule indicates that if a jail is in a city, then a hospital must also be in that city:
PT9-Oct1993_LGE-G4-srd2.png
As a complete aside, these two rules are kind of funny if you think about them literally: prisoners and college kids can’t be in the same city, but if you have a jail, you need a hospital (are they expecting a prison riot to break out?).

The fifth rule indicates that the universities are located in two cities that do not share a common border. More on this rule in a moment.

The sixth and final rule places two of the institutions: a university in city 3 and a jail in city 6. This can be diagrammed as:

PT9-Oct1993_LGE-G4-srd3.png

With these two institutions placed, a number of inferences can be made. With a jail in city 6, from the fourth rule there must be a hospital in city 6, and from the third rule there cannot be a university in city 6. There can also not be another jail in city 6 according to the second rule:

PT9-Oct1993_LGE-G4-srd4.png

With a university in city 3, from the fifth rule we can infer that there are no universities in cities 1, 4, and 5 (nor can there be another university in city 3 according to the second rule). Combining these inferences with the information above, we have one more university to place, but that university cannot be in cities 1, 3, 4, 5, or 6. Thus, the second university must be in city 2:

PT9-Oct1993_LGE-G4-srd5.png

Finally, from the third rule, we can eliminate jails from cities 2 and 3, both of which contain universities:

PT9-Oct1993_LGE-G4-srd6.png

The above diagram contains all of the institutions that can be placed, and all of the Not Laws that follow from the rules. At this point, there are only three hospitals and one jail that remain to be placed. Of course, the jail must be placed in a city with a hospital, and because the jail cannot be placed in cities 2, 3, and 6, we can infer that a jail and a hospital will be placed in city 1, 4, or 5:
PT9-Oct1993_LGE-G4-srd7.png
The remaining two hospitals have no restrictions, and they can be located within any city.

Thus, the final diagram for the game is:

PT9-Oct1993_LGE-G4-srd8.png
You do not have the required permissions to view the files attached to this post.
 Earbuddy
  • Posts: 2
  • Joined: Aug 18, 2011
|
#1526
The diagram given was slightly misleading by having the two rotated squares in the centers. (the diagram leads to confusion with rule 5 about the universities and boundaries)

And I would like clarification on the first rule please, which states,
"No institution is in more than one of the cities."

Thanks in Advance.
User avatar
 Dave Killoran
PowerScore Staff
  • PowerScore Staff
  • Posts: 5852
  • Joined: Mar 25, 2011
|
#1527
Well, the diagram given is a replica of the one used by LSAC, so what you are seeing is what people who took that test actually saw. The reason those squares are there is to make sure that there is no misinterpretation of the fifth rule about shared boundaries. The test makers intended that rule to apply to city pairs such as 1 and 2;1 and 3; 2 and 4; 3 and 4, etc. Without those squares, cities such as 1 and 4, and 2 and 3 would have a common boundary point.

The first rule is a cleanup rule that eliminates a number of possibilities. Basically, what the rule does is eliminate the possibility that an institution straddles the line. So, an institution can't be on the line between 1 and 3, and thereby qualify as being "in both."

Please let me know if that helps. Thanks!
 Earbuddy
  • Posts: 2
  • Joined: Aug 18, 2011
|
#1529
Thanks Dave,

That clears it up for me.
 hlg0035
  • Posts: 6
  • Joined: Apr 26, 2013
|
#9068
I do not understand the answers to question #s 22 and 23, will someone please explain?
User avatar
 Dave Killoran
PowerScore Staff
  • PowerScore Staff
  • Posts: 5852
  • Joined: Mar 25, 2011
|
#9077
Hi Hannah,

In order to help you out here, could you first give us an idea of the inferences you made during the setup? I'll assume you used the diagram they give in the scenario for your setup, but where did you go from there?

Thanks!
 hlg0035
  • Posts: 6
  • Joined: Apr 26, 2013
|
#9245
David,

I did not really make any inferences. I just used what was given in the rules, maybe that's my problem?

I used the set up that was given in the question.
User avatar
 Dave Killoran
PowerScore Staff
  • PowerScore Staff
  • Posts: 5852
  • Joined: Mar 25, 2011
|
#9263
Hi Hannah,

Thanks for the reply. Ok, your response helps. So, once you read through the rules and placed the U in city 3 and the J in city 6, you were then on to the questions. I think when you jumped right to the questions, you may have missed a few key points, and I'll outline those in a series of questions for you to consider when you look at this game again:
  • 1. Take a look again at the rules addressing the universities. What are the possibilities for the other university in this game?

    2. Turning to the jails and hospitals, the game leaves three hospitals and one jail unplaced. That jail must be accompanied by a hospital, so in effect you have the following left to place: JH, H, and H. Take a moment and consider where that JH block can be placed--what cities are available to it? The answer to this question plays a role in solving both questions #22 and #23.
Please take a look at those, and then let me know how it goes. Thanks!
 reop6780
  • Posts: 265
  • Joined: Jul 27, 2013
|
#11950
What kind of question is this one ?

I couldn't sort this question into either grouping or linear.

Get the most out of your LSAT Prep Plus subscription.

Analyze and track your performance with our Testing and Analytics Package.